Last visit was: 25 Apr 2024, 08:58 It is currently 25 Apr 2024, 08:58

Close
GMAT Club Daily Prep
Thank you for using the timer - this advanced tool can estimate your performance and suggest more practice questions. We have subscribed you to Daily Prep Questions via email.

Customized
for You

we will pick new questions that match your level based on your Timer History

Track
Your Progress

every week, we’ll send you an estimated GMAT score based on your performance

Practice
Pays

we will pick new questions that match your level based on your Timer History
Not interested in getting valuable practice questions and articles delivered to your email? No problem, unsubscribe here.
Close
Request Expert Reply
Confirm Cancel
SORT BY:
Kudos
Math Expert
Joined: 02 Sep 2009
Posts: 92914
Own Kudos [?]: 618958 [25]
Given Kudos: 81595
Send PM
Most Helpful Reply
User avatar
Manager
Manager
Joined: 03 Oct 2014
Posts: 115
Own Kudos [?]: 96 [10]
Given Kudos: 89
Location: India
Concentration: Operations, Technology
GMAT 1: 720 Q48 V40
WE:Engineering (Aerospace and Defense)
Send PM
Math Expert
Joined: 02 Sep 2009
Posts: 92914
Own Kudos [?]: 618958 [9]
Given Kudos: 81595
Send PM
General Discussion
avatar
SVP
SVP
Joined: 27 Dec 2012
Status:The Best Or Nothing
Posts: 1562
Own Kudos [?]: 7208 [3]
Given Kudos: 193
Location: India
Concentration: General Management, Technology
WE:Information Technology (Computer Software)
Send PM
Re: Mark bought shares for a total value of $1000. Next day, the price of [#permalink]
2
Kudos
1
Bookmarks
Answer = B = 11

Price/share ................. No of shares ................. Total

x ................................. \(\frac{1000}{x}\) .......................... 1000 (Let x = price per share)

\(\frac{9x}{10}\) .................... \(\frac{1000}{x}\) ........................... 900 (Price of shares reduced by 10% will affect price per share)

\(\frac{9x}{10}\) .................. \(\frac{1000}{x} + 10\) ..................... 1000 (Reduced price per share would give 10 more shares in the same amount)

Setting up the equation

\((\frac{9x}{10})(\frac{1000}{x} + 10) = 1000\)

\(x = \frac{1000}{9} - 100 = \frac{100}{9} = 11.111\)

Answer = B
avatar
Intern
Intern
Joined: 12 Feb 2020
Posts: 6
Own Kudos [?]: 2 [1]
Given Kudos: 16
Send PM
Re: Mark bought shares for a total value of $1000. Next day, the price of [#permalink]
1
Bookmarks
Bunuel wrote:
Mark bought shares for a total value of $1000. Next day, the price of shares is reduced by 10%. With the same $1000, he could now buy 10 more shares. Which of the following is closest to the original price per share?

A. $10
B. $11
C. $12
D. $13
E. $14

Kudos for a correct solution.


Suppose that number of shares is n
Suppose that price per share is p

1st day:
n*p=1000 (1)

2nd day:
Price per share is 0.9p
Number of shares is 10+n
0.9p * (10+n) =1000
9p + 0.9n*p = 1000 (2)

Replace (1) in (2):
9p + 900 = 1000
9p = 100

p = 11
GMAT Club Legend
GMAT Club Legend
Joined: 03 Oct 2013
Affiliations: CrackVerbal
Posts: 4946
Own Kudos [?]: 7626 [1]
Given Kudos: 215
Location: India
Send PM
Re: Mark bought shares for a total value of $1000. Next day, the price of [#permalink]
1
Kudos
Top Contributor
Lets use a strategy to solve this question under 2min.

For two quantities that are inversely proportional,

(i)an increase of x/y in one causes a corresponding decrease of x/(x +y) in the other

and

(ii)a decrease of x/y in one causes an increase of x/(y-x) in the other.

Price and consumption when expenditure is constant are inversely related.

The price of shares is reduced by 10% = 1/10 as a fraction with x =1 and y=10

Thus corresponding increase in quantity purchases is x/(y-x) = 1/9

1/9 is equivalent to 10 shares

=>1 or the original number of shares he bought = 9*10 = 90 shares

90 shares were bought for a total value of $1000

Price of 1 share = 1000/90 = 11(approx)

(option b )

D.S
GMAT SME
avatar
Intern
Intern
Joined: 07 Jun 2013
Posts: 8
Own Kudos [?]: 10 [0]
Given Kudos: 14
Send PM
Re: Mark bought shares for a total value of $1000. Next day, the price of [#permalink]
Price x Quantity = Amount(constant)
Since price is reduced to 9/10, so quantity will be increased by reversed amount 10/9 to make the amount constant.
Given increase in number of shares,i.e. 1/9 of Q is 10. so Quantity=90.
Price per share = Amount /quantity= 1000/90=11.11

B
Manager
Manager
Joined: 22 Jan 2014
Posts: 132
Own Kudos [?]: 244 [0]
Given Kudos: 212
WE:Project Management (Computer Hardware)
Send PM
Re: Mark bought shares for a total value of $1000. Next day, the price of [#permalink]
Bunuel wrote:
Mark bought shares for a total value of $1000. Next day, the price of shares is reduced by 10%. With the same $1000, he could now buy 10 more shares. Which of the following is closest to the original price per share?

A. $10
B. $11
C. $12
D. $13
E. $14

Kudos for a correct solution.


1000/(0.9x) - 1000/x = 10

let 1000/x = k

k/0.9 - k = 10
=> k = 90

so 1000/x = 90
=> x = 11.11 or 11
avatar
Intern
Intern
Joined: 25 Mar 2015
Posts: 8
Own Kudos [?]: 1 [0]
Given Kudos: 10
Schools: ISB '16
Send PM
Re: Mark bought shares for a total value of $1000. Next day, the price of [#permalink]
PX = 1000...........(a)

(0.9P)(X+10) = 1000......(b)

0.9PX + 9P = PX...
0.1PX = 9P...
X = 90...
Substitute in original...P = 11.1111
Senior Manager
Senior Manager
Joined: 05 Sep 2016
Status:DONE!
Posts: 274
Own Kudos [?]: 101 [0]
Given Kudos: 283
Send PM
Re: Mark bought shares for a total value of $1000. Next day, the price of [#permalink]
The following equation(s) can be derived from the prompt:

P = price
N = number of stock

(1) PN = 1000
(2) (P-0.10P)(N+10) = 1000

Plug (1) into (2) and you'll find P = 11.111....

Thus, B is the correct answer.
Manager
Manager
Joined: 14 Aug 2019
Posts: 159
Own Kudos [?]: 88 [0]
Given Kudos: 256
Location: Bangladesh
GPA: 3.41
Send PM
Re: Mark bought shares for a total value of $1000. Next day, the price of [#permalink]
Bunuel wrote:
Mark bought shares for a total value of $1000. Next day, the price of shares is reduced by 10%. With the same $1000, he could now buy 10 more shares. Which of the following is closest to the original price per share?

A. $10
B. $11
C. $12
D. $13
E. $14

Kudos for a correct solution.

Price = X
Quantity = Y
Hence, 1000 = XY
ATQ,
Price = .9X
Quantity = Y+10
Now,
XY = .9X*(Y+10)
XY = .9XY + 9X
Y = .9Y + 9Y
Y = 90
We supposed, XY = 1000 and we learned, Y =90
X*90 = 1000 => X = Price = 1000/90 = $11.11 = $11 Approx.

Ans. B
User avatar
Non-Human User
Joined: 09 Sep 2013
Posts: 32674
Own Kudos [?]: 821 [0]
Given Kudos: 0
Send PM
Re: Mark bought shares for a total value of $1000. Next day, the price of [#permalink]
Hello from the GMAT Club BumpBot!

Thanks to another GMAT Club member, I have just discovered this valuable topic, yet it had no discussion for over a year. I am now bumping it up - doing my job. I think you may find it valuable (esp those replies with Kudos).

Want to see all other topics I dig out? Follow me (click follow button on profile). You will receive a summary of all topics I bump in your profile area as well as via email.
GMAT Club Bot
Re: Mark bought shares for a total value of $1000. Next day, the price of [#permalink]
Moderators:
Math Expert
92914 posts
Senior Moderator - Masters Forum
3137 posts

Powered by phpBB © phpBB Group | Emoji artwork provided by EmojiOne